PROOFS OF IRRATIONALITY
NEIL MAKUR
Abstract. We start by looking at some basic properties regarding operations with rational
and irrational numbers. We then go on to show that certain radicals are irrational. Next,
we state and prove a criterion for irrationality and use it to prove that e is irrational. After
this, we look at alternate proofs of this fact, and show that e
r
is irrational for all nonzero
rationals r. Then, we see that π and π
2
are irrational, before going on to prove the same
for cos(1). We look at other irrational values related to the trigonometric functions next,
and prove that ln(r) is irrational for positive rational r 6= 1. We follow this up by looking
at other logarithms, and proving that cosh(1) is irrational. We then discuss transcendence,
and use the Lindemann–Weierstrass theorem to reprove many of our earlier statements. We
end with posing open problems in irrationality.
1. Closure of Irrational Numbers
In this section, we shall look at some basic theorems regarding operations with rational
and irrational numbers. All numbers in this paper will be assumed to be real unless they
are defined during our discussion of transcendence. We will also assume all denominators to
be nonzero.
Theorem 1.1. If r Q, and a 6∈ Q, then the following are true.
(1) r + a 6∈ Q.
(2) ra 6∈ Q.
(3) a 6∈ Q.
(4) a
1
6∈ Q.
Proof. Our methods for all of these will be proof by contradiction.
(1) If we assume that r + a Q, we will let it be r
0
Q. Then, a = r
0
r. Because Q
is additively closed, this means that a Q, a contradiction.
(2) If we assume that ra Q, we will let it be r
0
Q. Then, a =
r
0
r
. Because Q is
multiplicatively closed, this means that a Q, a contradiction.
(3) If we assume that a Q, we will let it be
p
q
Q. Then, a =
p
q
, meaning that it
is rational, which is a contradiction.
(4) If we assume that a
1
Q, we will let it be
p
q
Q. Then, a =
q
p
, meaning that it is
rational, which is a contradiction.
2. Irrational Radicals
2.1. Square Roots of Primes. It is well known that
2 is irrational. We will present the
classic proof here.
Date: July 2020.
1
2 NEIL MAKUR
Proposition 2.1.
2 is irrational.
Proof. Let us assume that
2 =
a
b
, where a and b are relatively prime integers. Then, 2 =
a
2
b
2
,
or 2b
2
= a
2
. Now, if a was odd, then a
2
= (2k + 1)
2
= 4k
2
+ 4k + 1 is also odd, so a must
be even. We will write a = 2k. Now, we have that 2b
2
= (2k)
2
= 4k
2
. Dividing by 2 gives
b
2
= 2k
2
. Therefore, b must be even. However, this contradicts our assumption that a and
b are relatively prime. Therefore,
2 is irrational.
We can also show this for
3.
Proposition 2.2.
3 is irrational.
Proof. Let us suppose that
3 =
a
b
where a and b are relatively prime integers. We can say
that 3b
2
= a
2
. Now, if we assume that 3 - a, then (as 3|a
2
), 3|a, which is a contradiction.
Therefore, we can write a = 3k, and get 3b
2
= (3k)
2
= 9k
2
. We can divide by 3 to get
b
2
= 3k
2
, so we can write b = 3l. This contradicts our assumption that a and b are relatively
prime, so
3 is irrational.
We can generalize this proofs easily to any prime. We shall first prove a lemma.
Lemma 2.3. If p|a
2
for a prime p, then p|a.
Proof. Let us suppose that p - a. Because p|a · a, and p - a, we have that p|a. This is a
contradiction, and so p|a.
Theorem 2.4. If p is a prime, then
p is irrational.
Proof. Let us suppose that
p =
a
b
where a and b are relatively prime integers. Then,
pb
2
= a
2
. We write a = pk to get pb
2
= (pk)
2
= p
2
k
2
. Dividing by p gives b
2
= pk
2
, and so
we can write b = pl, meaning that a and b are not relatively prime, which is a contradiction.
Therefore,
p 6∈ Q.
2.2. More Interesting Square Roots. We want to show that other roots are irrational.
We will show this for
6 below.
Proposition 2.5.
6 is irrational.
Proof. Let us suppose that
6 =
a
b
where a and b are relatively prime integers. Then,
6b
2
= a
2
. We can write a = 2k to get 6b
2
= 4k
2
. Thus, 3b
2
= 2k
2
, so we can write b = 2l.
This contradicts our assumption that gcd(a, b) = 1, so
6 6∈ Q.
Note that we could have said a = 3k, and used this to show that
6 is irrational. We can
generalize the proof of this.
Theorem 2.6. If p and q are distinct primes, then
pq is irrational.
Proof. Let us suppose that
pq =
a
b
where a and b are integers with gcd(a, b) = 1. Then,
pqb
2
= a
2
, so we can write a = pk to get pqb
2
= p
2
k
2
. Therefore, qb
2
= pk
2
, and (as q 6= p is
prime) we can say b = pl. This means that gcd(a, b) 6= 1, which is a contradiction.
PROOFS OF IRRATIONALITY 3
2.3. Sums of Square Roots. We want to show that sums of square roots are also irrational.
We will prove this for an example first.
Proposition 2.7.
2 +
3 is irrational.
Proof. Let us suppose that
2 +
3 =
a
b
for integers a and b such that
a
b
is fully reduced.
Then,
2 +
3
2
=
a
2
b
2
. This is equal to
2
2
+ 2
2
3 +
3
2
= 2 + 2
6 + 3 = 5 + 2
6.
Thus,
6 =
a
2
5b
2
2b
2
, which means that
6 Q. This is a contradiction, so
2 +
3 6∈ Q.
We can now see how to easily generalize this proof.
Theorem 2.8. If p and q are distinct primes, then
p +
q is irrational.
Proof. Let us suppose that
p +
q =
a
b
, where we have completely reduced fraction
a
b
with
a, b Z. Then,
p +
q
2
= p+q+2
pq =
a
2
b
2
. However, this implies that
pq is rational,
which is a contradiction. Therefore,
p +
q is irrational.
2.4. More General Roots. We will show that 2
1/n
is irrational for integers n 2.
Theorem 2.9. 2
1/n
is irrational for n Z 2.
Proof. If we assume that 2
1/n
=
a
b
where a, b Z are relatively prime, then we can say
2b
n
= a
n
. Thus, we can write a = 2k and derive b
n
= 2
n1
k
n
. This means that b is also
even, creating a contradiction.
We can in fact prove that 2
a/b
is irrational for positive integers a and b such that gcd(a, b) =
1, with a 6= 0 and b 6= 1. To do this, we shall use the rational root theorem.
Lemma 2.10 (Rational Root Theorem). If a fully reduced fraction
p
q
is a root of a
n
x
n
+
a
n1
x
n1
+ ··· + a
1
x + a
0
, then p|a
0
, and q|a
n
.
Proof. Let us suppose that
p
q
is a root. Then, we can say a
n
p
n
q
n
+a
n1
p
n1
q
n1
+···+a
1
p
q
+a
0
= 0.
Multiplying by q
n
gives a
n
p
n
+ a
n1
p
n1
q + ···+ a
1
pq
n1
+ a
0
q
n
= 0. We can rearrange this
to get a
n
p
n
+ a
n1
p
n1
q + ··· + a
1
pq
n1
= a
0
q
n
. Note that the left hand side is divisible
by p. Because gcd(p, q) = 1, this means that p|a
0
. Similarly, we have that a
n1
p
n1
q + ···+
a
1
pq
n1
+ a
0
q
n
= a
n
p
n
. Because q divides the left hand side, it must divide a
n
.
Theorem 2.11. For a fully reduced fraction
a
b
, where b 6= 1 and a 6= 0, 2
a/b
is irrational.
Proof. We only need to prove this for
a
b
> 0, because the reciprocal will also be irrational.
Therefore, we will assume
a
b
> 0. We can see that 2
a/b
is a root of x
b
2
a
. Therefore,
the rational roots are of the form ±2
k
for k a. Note that, if b is odd, the roots cannot
be negative, and if b is even, then if x is a root, so is x, so we only need to check the
positive cases. If this is a root, then (2
k
)
b
2
a
= 0. This means that a = kb, or that
a
b
= k,
contradicting the assumption that
a
b
is a fully reduced fraction with b 6= 1.
3. An Irrationality Criterion
Here we will prove the following theorem.
Theorem 3.1. A real number α is irrational if and only if, for any ε > 0, there exist integers
p and q such that
0 < |qα p| < ε.
4 NEIL MAKUR
3.1. The Dirichlet Approximation Theorem. To prove our desired criterion, we need
to use the Dirichlet Approximation Theorem. We will state and prove it here.
Theorem 3.2 (Dirichlet Approximation Theorem). Let N be a positive integer, and let α
be any real number. Then, there exist integers p and q with 1 q N such that
|qα p| <
1
N
.
Proof. We will consider the set A = {a
0
, a
1
, . . . , a
N
} where we define a
i
= bc. Clearly,
A [0, 1). Because |A| = N + 1, there is some positive k N so that
k1
N
,
k
N
contains two
elements of A. We will call these two elements a
i
and a
j
where i > j. Thus, |a
i
a
j
| <
1
N
.
Using our definition of a
i
, this means that |bcjα+bjαc| <
1
N
. We can rearrange this
to get |(i j)α (bc bjαc) | <
1
N
. Thus, we can choose q = i j, and p = bc bjαc
to prove the theorem.
3.2. The Irrationality Criterion. Now that we have proven the Dirichlet Approximation
Theorem, we are ready to prove the irrationality criterion.
Proof of Theorem 3.1. Let us suppose that α 6∈ Q, and let ε > 0. We shall choose N such
that
1
N
< ε. By the Dirichlet Approximation Theorem, there are integers p and q with
1 q N such that |qα p| <
1
N
< ε. Because α is irrational, this must be nonzero. Let
us now assume that α Q. We will write α =
r
s
for relatively prime integers r and s. Now,
if we have
p
q
6=
r
s
, where p, q Z, then
r
s
p
q
=
qr ps
qs
1
qs
.
Multiplying by q gives
|qα p|
1
s
.
Thus, if we choose
1
ε
> s, we get |qα p|
1
s
> ε. The only case we have disregarded is if
p
q
=
r
s
. In this case, |qα p| = 0. Thus, if α is rational, there are not integers p and q such
that 0 < |qα p| < ε for all ε > 0.
3.3. Applications of the Irrationality Criterion. We shall use the irrationality criterion
to show that e is irrational.
Theorem 3.3. e is irrational.
Proof. We know that e =
P
n=0
1
n!
. We will let N Z
>0
such that
1
N
< ε. We will let
q = N!, and p be
P
N
n=0
N!
n!
. Then,
|qe p| =
N!
X
n=0
1
n!
N!
N
X
n=0
1
n!
= N!
X
n=N+1
1
n!
=
X
n=N+1
1
(N + 1)(N + 2) ···(n 1)(n)
PROOFS OF IRRATIONALITY 5
X
n=N+1
1
(N + 1)
nN
.
Now, we may use the fact that this is a geometric series to get that this is equal to
1
N + 1
·
1
1 1/(N + 1)
.
This is equal to
1
N + 1
·
N + 1
N
=
1
N
< ε.
We can see that |qe p| 6= 0, so, by Theorem 3.1, e is irrational.
Theorem 3.1 can also be used to show that ζ(3) is irrational. This proof can be found
in [RSb, Chapter 4].
4. e and its Powers
4.1. Alternate Proofs of e’s Irrationality. We have already shown that e is irrational
using the irrationality criterion. We will present two alternate proofs here.
Second Proof of Theorem 3.3. We know that e =
P
n=0
1
n!
. If we assume that e =
a
b
for
relatively prime integers a and b, then we can say that n!be = n!a for every n > 0. We can
see that our right hand side is an integer. However, our left side is equal to
b
n
X
k=0
n!
k!
+ b
X
k=n+1
n!
k!
.
We can see, however, that the second sum is equal to
b
(n + 1)
+
b
(n + 1)(n + 2)
+ ··· .
We can see that this sum is greater than
b
n+1
. We can also see that, by comparison with
geometric series, that this is less than
b
n
. Thus, for large n, this is not an integer, which is a
contradiction.
Third Proof of Theorem 3.3. Let us assume that e =
p
q
where p and q are relatively prime
integers. Then, we can write
p
q
=
1
0!
+
1
1!
+
1
2!
+ ··· +
1
q!
+ ··· .
Now, we will rearrange this to get
p
q
1
0!
1
1!
···
1
q!
=
1
(q + 1)!
+
1
(q + 2)!
+ ··· .
We can multiply by q! to get
pq!
q
q!
0!
q!
1!
···
q!
q!
=
q!
(q + 1)!
+
q!
(q + 2)!
+ ··· .
6 NEIL MAKUR
It is easy to see that the left hand side is an integer. We can also see, by comparison with
geometric series, that our right hand side is less than
1
q
. It is also clearly positive. Thus,
this means that there is an integer n such that 0 < n <
1
q
, which is a contradiction.
Remark 4.1. It is not hard to see that these proofs are very similar. One can in fact modify
one proof to obtain the other.
4.2. The Irrationality of e
2
. To show that e
2
is irrational, we will use a similar technique
that we used for e.
Theorem 4.2. e
2
is irrational.
Proof. If we assume that e
2
=
a
b
where a, b Z and gcd(a, b) = 1, then be = ae
1
. We can
use the series e =
P
k=0
1
k!
and e
1
=
P
k=0
(1)
k
k!
, and multiply by n!. We can see that n!be
has an integer part, and a leftover term of
n!b
1
(n + 1)!
+
1
(n + 2)!
+ ···
.
We know that this term is between
b
n+1
and
b
n
. Similarly, n!ae
1
has a leftover part of
(1)
n+1
n!a
1
(n + 1)!
1
(n + 2)!
± ···
.
Now, if n is even, then this is larger than
a
n+1
, but smaller than
a
n + 1
+
a
(n + 1)
2
··· =
a
n + 1
1
1 + 1/(n + 1)
=
a
n + 1
1
1
n + 2
< 0.
However, this means that, for large n, n!ae
1
is slightly smaller than an integer, while n!be
is slightly larger than an integer. This is a contradiction, and so e
2
is irrational.
This idea can also be used to show that e
4
is irrational, see [AZHE10, Chapter 8].
4.3. Integer Powers of e. We shall prove the following theorem.
Theorem 4.3. For integers s 6= 0, e
s
is irrational.
To do this, we shall need to make use of a lemma.
Lemma 4.4. For some n 1, let
f
n
(x) =
x
n
(1 x)
n
n!
.
Then, the following are true.
(1) f
n
(x) can be written as
1
n!
P
2n
k=n
c
k
x
k
where c
k
Z.
(2) For 0 < x < 1, 0 < f
n
(x) <
1
n!
.
(3) For all k > 0, f
(k)
n
(0) and f
(k)
n
(1) are integers.
Proof. Part (1) is clear, and part (2) can be checked by basic differentiation, so we will prove
only part (3). Note that, f
(k)
n
(0) is 0 unless n k 2n. Because we are plugging in 0, we
only need to worry about the constant term. The constant term for these are
k!
n!
c
k
, which is
an integer in the range. Now, notice that f
n
(x) = f
n
(1 x), so f
(k)
n
(x) = (1)
k
f
(k)
n
(1 x),
which means that f
(k)
n
(1) = (1)
k
f
(k)
n
(0), which is an integer.
PROOFS OF IRRATIONALITY 7
We are now able to prove our theorem.
Proof of Theorem 4.3. Let us suppose that e
s
=
a
b
for relatively prime integers a and b. We
will let n be determined later, and we will let
F (x)
:
= s
2n
f(x) s
2n1
f
0
(x) + s
2n2
f
00
(x) ··· + f
(2n)
(x),
where we write f(x) for f
n
(x) from the above lemma. Because f
(k)
(x) = 0 for k > 2n, we
can write
F (x) = s
2n
f(x) s
2n1
f
0
(x) + s
2n2
f
00
(x) ··· .
This allows us to say
F
0
(x) = sF (x) + s
2n+1
f(x).
Now, let us look at
d
dx
(e
sx
F (x)). The product rule gives
d
dx
(e
sx
F (x)) = se
sx
F (x) + e
sx
F
0
(x) = se
sx
F (x) + e
sx
(sF (x) + s
2n+1
f(x))
= s
2n+1
e
sx
f(x).
This allows us to find
N
:
= b
Z
1
0
s
2n+1
e
sx
f(x)dx = b (e
sx
F (x)) |
1
0
= be
s
F (1) bF (0) = aF (1) bF (0).
Part (3) of our lemma implies that this is an integer. Part (2) of the lemma gives us bounds,
which allow us to say that
0 < N <
bs
2n
n!
Z
1
0
se
sx
dx =
bs
2n
n!
(e
s
1).
We can assume that s is positive, and the closure properties will cover the case of s < 0. We
will also let n be such that n! > as
2n+1
. Thus,
N <
bs
2n+1
e
s
n!
=
as
2n+1
n!
< 1.
This is a contradiction, and so e
s
is irrational.
4.4. Rational Powers of e. We have shown that for n Z\{0}, e
n
is irrational. For
n R\{0}, this is clearly false (for example, n = ln(2)). We wish to determine this for the
rationals.
Theorem 4.5. If r is a nonzero rational number, then e
r
is irrational.
Proof. We will write r =
p
q
for p, q Z and gcd(p, q) = 1. Let us suppose that e
p/q
=
a
b
for
relatively prime integers a and b. This means that e
p
=
a
q
b
q
. This means that e
p
is rational,
a contradiction.
8 NEIL MAKUR
5. Pi, Trigonometric, Logarithmic, and Hyperbolic Values
5.1. π and π
2
’s Irrationality. We will first prove the well-known theorem that π is irra-
tional.
Theorem 5.1. π is irrational.
Proof. We will let π =
a
b
where a and b are integers with gcd(a, b) = 1. We define
f(x)
:
=
x
n
(a bx)
n
n!
.
Note that we can make an analog of Lemma 4.4 into a lemma based around f(x). Proving
the exact details of this analog is left as an exercise for the reader. We will now define
F (x) = f(x) f
(2)
(x) + f
(4)
(x) ··· .
Notice that we can have this infinite series because the derivatives beyond 2n vanish. We
also have that F
00
(x) = F (x) + f(x). Basic calculus gives that
d
dx
(F
0
(x) sin(x) F (x) cos(x)) = F
00
(x) sin(x) + F
0
(x) cos(x) F
0
(x) cos(x) + F (x) sin(x)
= F
00
(x) sin(x) + F (x) sin(x) = (F (x) + f(x)) sin(x) + F (x) sin(x) = f(x) sin(x).
Thus,
Z
π
0
f(x) sin(x)dx = (F
0
(x) sin(x) F (x) cos(x)) |
π
0
= F (π) + F (0).
This is an integer. However, we can also establish a bound on this for 0 < x < π:
0 < f(x) sin(x) < f(x) <
π
n
a
n
2
2n
n!
.
Thus, our integral (which is an integer) is arbitrarily small for large enough n, and so we
have a contradiction. Thus, π is irrational.
Corollary 5.2. For positive integers n, π
1/n
is irrational.
Proof. Let us suppose that π
1/n
is rational, say
a
b
. Then, we have that
a
n
b
n
= π. This is a
contradiction, and so π
1/n
is irrational.
Now, we will do the same for π
2
. When we write f(x), we will mean f
n
(x) from Lemma
4.4.
Theorem 5.3. π
2
is irrational.
Proof. Let us assume π
2
=
a
b
where a, b Z, and are relatively prime. We define
F (x)
:
= b
n
π
2n
f(x) π
2n2
f
(2)
(x) + π
2n4
f
(4)
(x) ···
.
Note that this satisfies F
00
(x) = π
2
F (x) + b
n
π
2n+2
f(x). We can get that
d
dx
(F
0
(x) sin(πx) πF (x) cos(πx)) = F
00
(x) sin(πx) + π
2
F (x) sin(πx)
= b
n
π
2n+2
f(x) sin(πx) = π
2
a
n
f(x) sin(πx).
Therefore,
N
:
= π
Z
1
0
a
n
f(x) sin(πx)dx =
π
1
F
0
(x) sin(πx) F (x) cos(πx)
|
1
0
= F (1) + F(0).
PROOFS OF IRRATIONALITY 9
We can see that N Z. Now, let us choose n so large such that
2a
n
n!
< 1. Our bounds on
f(x) allow us to say that
0 < N = π
Z
1
0
a
n
f(x) sin(πx)dx < πa
n
1
n!
Z
1
0
sin(πx)dx =
2a
n
n!
< 1.
This is a contradiction.
5.2. The Irrationality of cos(1). Here we will prove that cos(1) is irrational using the
irrationality criterion.
Proposition 5.4. cos(1) is irrational.
Proof. We know that cos(1) =
P
n=0
(1)
n
(2n)!
. We will choose N such that
1
N
< ε. We will let
q = N! and p = N!
P
N
n=0
(1)
n
(2n)!
. Then, we have that
|q cos(1) p| =
N!
X
n=0
(1)
n
(2n)!
N!
N
X
n=0
(1)
n
(2n)!
=
X
n=N+1
(1)
n
N!
(2n)!
X
n=N+1
(1)
n
(N + 1)
2nN
X
n=N+1
1
(N + 1)
2nN
=
1
(N + 1)
N+2
·
1
1 1/(N + 1)
2
=
1
(N + 1)
N+2
·
(N + 1)
2
(N + 1)
2
1
=
1
N
·
1
(N + 1)
N
·
1
N + 2
<
1
N
< ε.
Therefore, by Theorem 3.1, cos(1) is irrational.
This proof can be easily modified to show that sin(1) is irrational, and is left as an exercise
to the reader.
5.3. Other Trigonometric Values. Many values of trigonometric functions, such as cos
π
9
and sin
π
18
are irrational. We will show these here.
Proposition 5.5. cos
π
9
is irrational.
Proof. By the triple angle formula, we have that cos
π
3
= 4 cos
3
π
9
3 cos
π
9
. Writing
x for cos
π
9
gives
1
2
= 4x
3
3x, or 8x
3
6x 1 = 0. The rational roots of these must be
±1, ±
1
2
, ±
1
4
, or ±
1
8
. Direct substitution gives that none of these are actual roots. Thus, as
cos
π
9
is a root of this, cos
π
9
is irrational.
We can prove that sin
π
18
is irrational 2 different ways.
Proposition 5.6. sin
π
18
is irrational.
First Proof of Proposition 5.6. From the triple angle formula for sin, we have that sin
π
6
=
3 sin
π
18
4 sin
3
π
18
, meaning that sin
π
18
is a root of
1
2
= 3x 4x
3
, or 8x
3
6x + 1 = 0.
We can once again use the rational root theorem to show that this has no rational roots,
and so sin
π
18
is irrational.
Second Proof of Proposition 5.6. We have that, from the cosine double angle formula, cos
π
9
=
1 2 sin
2
π
18
. Thus, if sin
π
18
were rational, then cos
π
9
would be too. This is a contra-
diction, so sin
π
18
must be irrational.
10 NEIL MAKUR
5.4. Inverse Trigonometric Functions. We will prove a result regarding the irrationality
of inverse trigonometric functions.
Theorem 5.7. For every odd integer n 3,
1
π
cos
1
1
n
is irrational.
Recall that, letting n = 9, this is useful in the solution of Hilbert’s Third Problem. In the
proof, we shall make use of a lemma.
Lemma 5.8. cos((k + 1)ϕ) = 2 cos(kϕ) cos(ϕ) cos((k 1)ϕ).
Proof. Recall that cos(α + β) = cos(α) cos(β) sin(α) sin(β), and that cos(α β) =
cos(α) cos(β) + sin(α) sin(β). Adding these gives that
cos(α + β) + cos(α β) = cos(α) cos(β) sin(α) sin(β) + cos(α) cos(β) + sin(α) sin(β)
= 2 cos(α) cos(β).
If we substitute θ = α + β and φ = α β, then α =
θ+φ
2
, and β =
θφ
2
. This gives the
formula
cos(θ) + cos(φ) = 2 cos
θ + φ
2
cos
θ φ
2
.
Now, if we let θ = (k + 1)ϕ, and φ = (k 1)ϕ, we get that
cos((k + 1)ϕ) + cos((k 1)ϕ) = 2 cos(kϕ) cos(ϕ),
which is the same thing as saying that
cos((k + 1)ϕ) = 2 cos(kϕ) cos(ϕ) cos((k 1)ϕ).
Proof of Theorem 5.7. We shall define ϕ
n
= cos
1
1
n
. We wish to show that
cos(kϕ
n
) =
A
k
n
k
,
where A
k
is an integer such that n - A
k
. This can be done with induction on k. For k = 0, 1,
this is true with A
k
= 1. We can use our formula to show this for k + 1:
cos((k + 1)ϕ
n
) = 2
1
n
A
k
n
k
A
k1
n
k1
=
2A
k
nA
k1
n
k+1
:
=
A
k+1
n
k+1
.
It is clear that, if n - A
k
, then n - A
k+1
. Now, let us assume that
1
π
ϕ
n
=
k
l
for relatively
prime integers k and l. Then, we have that lϕ
n
= kπ. Taking the cosine of both sides gives
A
l
n
l
= cos(lϕ
n
) = cos(kπ) = ±1. Therefore,
n
l
= ±A
l
. It is clear that l 6= 0, and that this
doesn’t work for l = 1. Therefore, we can assume that l 2. In this case, n|
n
l
. From the
fact that
n
l
|A
l
, we have that n|A
l
, which is a contradiction.
PROOFS OF IRRATIONALITY 11
5.5. Irrationality of Natural Logarithms. It is well known that ln(2) is irrational. We
shall prove this here.
Proposition 5.9. ln(2) is irrational.
Proof. Let us assume that ln(2) =
a
b
with a, b Z and gcd(a, b) = 1. Then, 2 = e
a/b
.
However, we know that e
a/b
must be irrational, so this is a contradiction.
It is clear that there is nothing about 2 in this situation, and so we can prove this for
(almost) any rational number.
Theorem 5.10. If r Q
>0
\{1}, then ln(r) is irrational.
Proof. If we assume that ln(r) =
a
b
where a and b are integers with gcd(a, b) = 1, then we can
say that r = e
a/b
. However, this means that r must be irrational, which is a contradiction.
5.6. Irrationality of Logarithms with Integer Bases. A well known irrational number
is log
2
(3). We shall prove that this is irrational.
Proposition 5.11. log
2
(3) is irrational.
Proof. Let us suppose that log
2
(3) =
a
b
with a, b Z and gcd(a, b) = 1. Then, 3 = 2
a/b
, or
3
b
= 2
a
. By the Fundamental Theorem of Arithmetic, this is only possible if a = b = 0.
Therefore, log
2
(3) is irrational.
It is clear that there is nothing special about 3 here, so we will prove this generally.
Proposition 5.12. If n Z is not a power of 2, then log
2
(n) is irrational.
Proof. Let us write n = 2
p
a such that a is odd. Now, let us assume that log
2
(n) =
b
c
where b
and c are relatively prime positive integers. Then, 2
b
= 2
pc
a
c
. This is only possible if either
a = 1, or c = 0, which contradict our initial conditions. Therefore, log
2
(n) is irrational.
We will show that the same is true for 10.
Proposition 5.13. If n Z is not a power of 10, then log
10
(n) is irrational.
Proof. Let us write n = 2
p
1
5
p
2
a such that a is relatively prime to 10. If we assume that
log
10
(n) =
b
c
where b and c are relatively prime positive integers, then we have that 10
b
=
2
p
1
c
5
p
2
c
a
c
. If a 6= 1, then this is impossible. If a = 1, then we have that p
1
c = p
2
c = b.
However, this means that p
1
= p
2
, or that n is a power of 10, which is a contradiction.
This may lead us to believe that this is true for every integer. However, this is false.
Counterexample. log
8
(16) =
4
3
.
5.7. Irrationality of cosh(1). Recall that cosh(x) is defined as
cosh(x) =
e
x
+ e
x
2
,
and that sinh(x) is defined as
sinh(x) =
e
x
e
x
2
.
Remark 5.14. If the reader is familiar with the complex definitions of sin(x) and cos(x), they
may notice that cosh(t) = cos(it), and that sinh(t) = i sin(it), where i =
1.
12 NEIL MAKUR
Thus, cosh(1) =
e+e
1
2
. We wish to show that this is irrational.
Proposition 5.15. cosh(1) is irrational.
Proof. We have that
cosh(1) =
e + e
1
2
=
1
2
X
n=0
1
n!
+
X
n=0
(1)
n
n!
!
=
1
2
X
n=0
1 + (1)
n
n!
!
=
1
2
2
0!
+
2
2!
+
2
4!
+ ···
=
1
0!
+
1
2!
+
1
4!
+ ··· =
X
n=0
1
(2n)!
.
To show that this is irrational, we shall use our irrationality criterion. Let us choose N such
that
1
N
< ε, and let q = N!, p =
P
N
n=0
N!
(2n)!
. Therefore,
|q cosh(1) p| =
N!
X
n=0
1
(2n)!
N!
N
X
n=0
1
(2n)!
=
X
n=N+1
N!
(2n)!
=
X
n=N+1
1
(N + 1)(N + 2) ···(2n)
X
n=N+1
1
(N + 1)
2nN
=
1
(N + 1)
N+2
·
1
1 1/(N + 1)
2
=
1
(N + 1)
N+2
·
(N + 1)
2
(N + 1)
2
1
=
1
N
·
1
(N + 1)
N
·
1
N + 2
<
1
N
< ε.
Thus, by Theorem 3.1, cosh(1) is irrational.
This proof is easily modified to show that sinh(1) is irrational, and we leave it as an
exercise to the reader.
6. Transcendence
In this paper, we have looked at irrationality. We will now turn to transcendence.
Definition 6.1. A number α is said to be algebraic if it is the root of some nonzero poly-
nomial with integer (or equivalently, rational) coefficients.
Example. The number i =
1 is a root of x
2
+ 1, and is therefore algebraic.
Example. A rational number
a
b
is a root of x
a
b
(and bx a), and is therefore algebraic.
Example.
2 is a root of x
2
2, and is therefore algebraic.
Remark 6.2. The algebraic numbers form a field (i.e. they are closed under addition, sub-
traction, multiplication, and division). The simplest proof of this requires Abstract Algebra.
We refer the reader to [RSa] to obtain the necessary background. If α and β are algebraic
over Q, then we have that [Q(α, β) : Q] is finite. We shall define this number to be n.
Therefore, the numbers α + β, (α + β)
2
, . . . , (α + β)
n+1
are linearly dependent. This gives us
a nonzero polynomial with root α + β. The same is true for αβ and α
1
.
Definition 6.3. A number is said to be transcendental if it is not algebraic.
Our main tool will be the Lindemann–Weierstrass theorem.
Theorem 6.4 (The Lindemann–Weierstrass Theorem). If u 6= 0 is algebraic, then e
u
is
transcendental.
PROOFS OF IRRATIONALITY 13
The proof of this theorem is beyond the scope of this paper. We refer the reader to [Wik20]
for a proof.
This theorem will allow us to easily prove many transcendence (and hence, irrationality)
results.
Corollary 6.5. e is transcendental.
Proof. Take u to be 1.
Corollary 6.6. e
r
is transcendental for rational r.
Proof. Rational numbers are algebraic. Therefore, take u = r.
Corollary 6.7. π is transcendental.
Proof. Let us assume that π is algebraic. Then, we have that is algebraic. This means
that, by Theorem 6.4, e
= 1 is transcendental, a contradiction.
Corollary 6.8. π
n
is transcendental for nonzero integers n.
Proof. Let us assume that n > 0. Then, if π
n
is a root of p(x), π is a root of p(x
n
), which
is a contradiction. Now, because the algebraic numbers form a field, if π
n
is algebraic for
n > 0, then π
n
would be algebraic, which is a contradiction.
Corollary 6.9. π
a/b
is transcendental for rational
a
b
6= 0.
Proof. If π
a/b
were algebraic, then
π
a/b
b
= π
a
would also be algebraic. This is a contradic-
tion.
Corollary 6.10. sin(n) and cos(n) are transcendental for nonzero algebraic numbers n.
Proof. We have that e
in
= cos(n) + i sin(n). Now, we know that sin(n) =
p
1 cos
2
(n).
Thus, if cos(n) were transcendental, then so is cos
2
(n) and 1 cos
2
(n). This means that
p
1 cos
2
(n) = sin(n) is also transcendental. We can also swap sin(n) and cos(n) to get
that sin(n) is transcendental if and only if cos(n) is as well. Now, if sin(n) is algebraic, so
is cos(n), and therefore, cos(n) + i sin(n) is algebraic. Because we are assuming that n is
algebraic, this is a contradiction, and so sin(n) and cos(n) are transcendental.
Corollary 6.11. sin
1
(n) and cos
1
(n) are transcendental for algebraic nonzero n.
Proof. Let φ = sin
1
(n). Then, sin(φ) and cos(φ) are algebraic. If we assume that φ
is algebraic, then e
= cos(φ) + i sin(φ) is transcendental by the Lindemann–Weierstrass
Theorem. However, this is an algebraic number, so φ must be transcendental. The same is
true for φ
0
= cos
1
(n).
Corollary 6.12. cosh(n) and sinh(n) are transcendental for algebraic nonzero n.
Proof. Let us suppose that n is algebraic. Then, in is also algebraic. Thus, sin(in) =
i sinh(n) and cos(in) = cosh(n) are transcendental. Clearly, if i sinh(n) is transcendental, so
is sinh(n).
Corollary 6.13. ln(n) is transcendental for nonzero algebraic n.
Proof. Let us suppose that ln(n) is algebraic. Then, we have that e
ln(n)
= n is transcendental
by Theorem 6.4, which is a contradiction.
14 NEIL MAKUR
Another important theorem is the Gelfond–Schneider Theorem.
Theorem 6.14 (The Gelfond–Schneider Theorem). If a 6= 0, 1 is algebraic, and b is an
irrational algebraic number, than a
b
is transcendental.
For a proof, we refer the reader to [War12]. This allows us to prove a few more facts.
Corollary 6.15.
2
2
is transcendental.
Proof. Take a = b =
2 (which we already know is irrational).
Corollary 6.16. e
π
is transcendental.
Proof. We apply Euler’s Formula to get that
e
π
= e
i·
=
e
i
= (1)
i
,
so let a = 1 and b = i. Therefore, e
π
is transcendental.
7. Open Problems in Irrationality
Despite the fact that almost any person knows what an irrational number is, there are
still many questions about which numbers are irrational.
Question 7.1. Is e + π irrational?
Question 7.2. Is e · π irrational?
Question 7.3. Is π
e
irrational?
None of these answers are known, but are all believed to be “yes”. However, there is some
work done towards the solutions of the first two questions.
Proposition 7.4. At least one of e · π and e + π is irrational.
Proof. It is clear that both e and π are roots of (xe)(x π) = x
2
(e+ π)x +e·π. Because
both e and π are transcendental, at least one of e + π and e · π must be irrational.
References
[AZHE10] Martin Aigner, unter M Ziegler, Karl H Hofmann, and Paul Erdos. Proofs from the Book.
Springer, 2010.
[Niv61] Ivan Niven. Numbers: rational and irrational, volume 1. Random House New York, 1961.
[Niv85] Ivan Niven. Irrational numbers, volume 11. American Mathematical Soc., 1985.
[Niv00] Ivan Niven. A simple proof that π is irrational. In Pi: A Source Book, pages 276–276. Springer,
2000.
[RSa] Simon Rubinstein-Salzedo. Abstract Algebra. Self Published.
[RSb] Simon Rubinstein-Salzedo. Infinite Series. Self Published.
[War12] Matthijs Warrens. On irrational and transcendental numbers. 2012.
[Wik20] Wikipedia contributors. Lindemann–weierstrass theorem, 2020. [Online; accessed 30-July-2020 ].